PopularFX
Home Help Search Login Register
Welcome,Guest. Please login or register.
2024-04-19, 21:33:14
News: If you have a suggestion or need for a new board title, please PM the Admins.
Please remember to keep topics and posts of the FE or casual nature. :)

Pages: [1] 2 3
Author Topic: Smudge's Papers  (Read 12480 times)

Group: Moderator
Hero Member
*****

Posts: 1856
While searching for a paper I wrote years ago I cam across several ones of interest so I thought I would post them here.

Here is the first one which suggests that we could have a PM generator consisting of a PM rotating within a coil.  With the coil carrying load current it creates a magnetic field that the PM sees, and that applies drag torque to the rotating magnet.  My idea was to surround the magnet with a superconducting shorted turn that will divert the flux so that the magnet will now not see the flux and therefore will not endure drag torque.  I have since realized that the superconductor must not inhibit the magnetic field emanating from the magnet poles, so it can't be as shown in the paper, but it can be like two shorted turns one each side of the magnet.

I have just done some FEMM simulations where you can set up a diamagnetic material with a mu much less than unity (I used mu = 0.001) and that simulates the superconductor.  The first image below shows the Lenz field from the coil as directly applied to the magnet (the magnet's field is not shown in this view as that distorts the picture).  The torque as calculated by FEMM is positive, an arbitrary figure here.  The next image shows the effect of having superconducting material either side of the magnet, the Lenz field is diverted around the magnet.  I expected the torque in this situation to be much reduced, but to my surprise it actually reversed.   So it appears that we can make a generator where the Lenz flux from the load current creates a boost torque, not a drag torque.  Now that is something to think about.

Edit.  I should point out that I checked that the presence of the superconductors did not reduce the magnet's flux coupling to the coil, so it would still generate voltage.

Smudge 
   

Group: Professor
Hero Member
*****

Posts: 3359
Here is the first one which suggests that we could have a PM generator consisting of a PM rotating within a coil.
Rotating around which axis ?  Could you add some arrows to your diagrams ?

...and that applies drag torque to the rotating magnet.
Is the magnet itself electrically conductive ?
   

Group: Moderator
Hero Member
*****

Posts: 1856
Rotating around which axis ?  Could you add some arrows to your diagrams ?
OK, here are the same images with the rotation denoted, with the field arrows and the magnet poles marked.  Also I have added a 3D view of the model showing the dimensions I used to get those torque figures.
Quote
Is the magnet itself electrically conductive ?
I used the NdFeB 52 MGOe model available in FEMM, but I was only looking for some guidance as to the effect of shielding the magnet with superconducting material, so the conductivity of the magnet didn't come into it.  FEMM gives a snapshot and I chose the point where the magnet's coupling to the coil is going through zero, the output voltage and load current is maximum hence drag torque is maximum.  This was not a dynamic run, I just used 100 amp-turns in the coil as a load current.  Quite probably the conditions needed to get this would be an impossible rotor speed, but I was looking for the principle.  I was surprised to see the drag torque become a boost torque, but that may be a simulation artifact.  FEMM uses the Maxwell stress tensor for its torque and my reading on that subject tells me the stress tensor ought to be taken close to the surface of the rotating  magnetic object.  FEMM doesn't do that, it creates a stress tensor mask some distance from the surface.

My next step is to do some full dynamic runs using FEMM.  In keeping with the title of this thread I attach my paper describing how FEMM can be used for dynamic modelling.

Smudge
   

Group: Moderator
Hero Member
*****

Posts: 1856
Well I checked out the stress tensor mask used in FEMM for obtaining torque, and it is very susceptible to the mesh size.  When I increased the mesh (reduced from 2mm down to 1mm spacing) the reversed drag torque disappeared.  It became drag, not boost torque, of significant amplitude so my idea that the superconductor would shield the magnet didn't work.  To explore why, I put a thinner magnet inside the shield so that I could have an air space around the magnet (the stress tensor mask has to be through air).  I could then examine the torque on the magnet itself and low and behold the magnet was screened by the superconductors, there was little torque on the magnet.  The drag torque had all transferred to the superconducting screens.  That gives me a problem because when I consider the currents induced into the superconductor (or in the case of a highly diamagnetic material the alignment of the atomic dipoles therein that make it exclude the field) I can't see how they can carry that torque.  The stress tensor approach calculates torque as though the magnetic field itself carries that mechanical torque, but there is no matter there in that field.  In my mind the only place where the mechanical torque can materialize is on matter itself, or rather on the atomic dipoles within that matter.  I think I need to do more to see how the complex field pattern that exhibits torque via the stress tensor really does tell us the torque on those atomic dipoles.
Smudge
   

Group: Moderator
Hero Member
*****

Posts: 1856
Not sure if I have posted this before, but here is my explanation for the ball-bearing motor.  It relies on the bearing being of steel and therefore ferromagnetic where conduction electrons can be spin polarized.  Current passing through the ball transports angular momentum.

While on the subject of spin polarization here is another paper offering an interesting experiment that may offer OU potential.

Smudge
   

Group: Moderator
Hero Member
*****

Posts: 1856
Here's another paper that I partly wrote some time ago and have just completed.  It's for the TPU buffs as it describes how a TPU could get energy from the Earth's field, and in particular the Earth's scalar magnetic potential which has a huge value away from the equator.  It can explain a lot of the known facts about the Steven Marks TPU, like its gyroscopic feel, its inner coil (maybe not a coil) wound onto a cork former and it won't work upside down.  Doesn't fit with Marks 5KHz frequency though.  Enjoy!
Smudge 
   

Group: Moderator
Hero Member
*****

Posts: 1856
And now for something different.  This paper shows three images.  The first image shows a how two magnets where their axes are always parallel can have attraction and repulsion zones.  At the dividing line between these zones, the so called neutral line, the linear force is zero but there is a strong torque.  The second image shows how one magnet can follow a circular path where the linear forces provide a driving force around that path.  Of course it requires something to keep the axes parallel.  The final image shows the Ferris wheel motor where gravity is the force that keeps the axes parallel.  Worth building?
Smudge 
   

Group: Moderator
Hero Member
*****

Posts: 1856
This suggests that the Bearden MEG relies on magnetostriction of the metglas core coupled with the saturation of the inner laminations so as to make use of the Villari effect.   The core will have an ultrasonic resonance.
Smudge
   

Group: Experimentalist
Hero Member
*****

Posts: 765
Believing in something false doesn't make it true.
And now for something different.  This paper shows three images.  The first image shows a how two magnets where their axes are always parallel can have attraction and repulsion zones.  At the dividing line between these zones, the so called neutral line, the linear force is zero but there is a strong torque.  The second image shows how one magnet can follow a circular path where the linear forces provide a driving force around that path.  Of course it requires something to keep the axes parallel.  The final image shows the Ferris wheel motor where gravity is the force that keeps the axes parallel.  Worth building?
Smudge

Thanks for the interesting paper.  My testing confirms what you have posted.  I like your idea for the Ferris wheel version.  I am currently working on my own magnet motor idea.  If it doesn't work out I will probably attempt to build your Ferris wheel version.

Carroll


---------------------------
Just because it is on YouTube does not make it real.
   

Group: Tinkerer
Hero Member
*****

Posts: 1708
You’ve been doing my head in all afternoon with Ferris wheel structures smudge  O0 ;D ;D ;Di
   

Group: Moderator
Hero Member
*****

Posts: 1856
Being at the ripe old age of 86 and having to stay in the house until our government deems otherwise, I have been looking back at my past work.  Found this paper and I can't remember publishing it anywhere.  Wouldn't it be nice if someone could purchase room temperature superconductor material to give this a try?

Smudge
   

Group: Moderator
Hero Member
*****

Posts: 1856
Just to fill in time while being socially isolated I looked back at Howard Johnson's work.  Here is my critique of his vortex claims, they are not what they seem.  My message is to ignore his ideas about PM fields.

That is not to say that the classical model for a PM is necessarily correct under all circumstances.  In a metal magnet conduction electrons become spin polarized and add to the magnetization.  Being mobile within the magnet they endure magnetic forces that can cause them to dynamically congregate in certain areas, thus the overall magnetization is no longer spatially uniform within the magnet.  I can't find any evidence that this effect has ever been considered, and this could be the key to making magnet motors work.

Smudge
   
Group: Experimentalist
Hero Member
*****

Posts: 841
And now for something different.  This paper shows three images.  The first image shows a how two magnets where their axes are always parallel can have attraction and repulsion zones.  At the dividing line between these zones, the so called neutral line, the linear force is zero but there is a strong torque.  The second image shows how one magnet can follow a circular path where the linear forces provide a driving force around that path.  Of course it requires something to keep the axes parallel.  The final image shows the Ferris wheel motor where gravity is the force that keeps the axes parallel.  Worth building?
Smudge

Well it seemed to good to be true! So a bit red neck but here is my attempted build.

The magnets are 11mm X 38mm. The counterweight is a lead slug about 3/4 X 1 inch and weighs 82 grams.

It has problems at these dimensions as attract in is much stronger than repulse out. That and here we see that the counter weight is not strong enough to keep the rotor magnet level. A further complication is at speed the counter weight has a tendency to fly out.

Anyway all good fun, sorry my build didn't work.  :-[

Ron

https://youtu.be/eMGiCx4Yw0o
« Last Edit: 2020-04-04, 01:57:17 by ronee »
   

Group: Mad Scientist
Hero Member
*****

Posts: 546
hey Ronee

Vid says private

Mags
   
Group: Experimentalist
Hero Member
*****

Posts: 841
hey Ronee

Vid says private

Mags

Sorry about that, youtube have a new program out that has too many bells and whistles for me... so I set it to use unlisted on the old "classic program" Well turns out they have one more setting called "Publish" which on the old program was "Done" and I had not clicked on Publish...

Makes me wonder if they have a whole room full of people sitting around with the mandate of, "if it works, how can we f*ck it up?"

Ron
   

Group: Experimentalist
Hero Member
*****

Posts: 765
Believing in something false doesn't make it true.
Switch to Vimeo.  Much simpler to use and my videos seem to load faster there also.  I have a slow DSL service and the same video that loaded in 1 and 1/2 hours to Vimeo, I spent 4 trying to load to Youtube and then Youtube didn't "process" it, whatever that means.  You just load your video to Vimeo and then set private or public or only allow people with the link to see it.  So much simpler than Youtube.

Carroll


---------------------------
Just because it is on YouTube does not make it real.
   
Group: Experimentalist
Hero Member
*****

Posts: 841
Switch to Vimeo.  Much simpler to use and my videos seem to load faster there also.  I have a slow DSL service and the same video that loaded in 1 and 1/2 hours to Vimeo, I spent 4 trying to load to Youtube and then Youtube didn't "process" it, whatever that means.  You just load your video to Vimeo and then set private or public or only allow people with the link to see it.  So much simpler than Youtube.

Carroll

Thanks Carroll,

Youtube is becoming (has become) a POS. I have used it since day one and at that time the thing to do was be anonymous. My handle was thus a take off on "John Doe" as I used Ron with an H in "rohndoe"

Now when I click on my URL I get a "do you mean rondoe" ... I always think, where is the "FO" button
That and now there are fully 6 videos on rohndoe that are not mine... with no way to delete them!!!

https://www.youtube.com/results?search_query=rohndoe

So Vimeo is starting to look good... Thanks

Ron
   
Group: Experimentalist
Hero Member
*****

Posts: 841
Well it seemed to good to be true! So a bit red neck but here is my attempted build.

The magnets are 11mm X 38mm. The counterweight is a lead slug about 3/4 X 1 inch and weighs 82 grams.

It has problems at these dimensions as attract in is much stronger than repulse out. That and here we see that the counter weight is not strong enough to keep the rotor magnet level. A further complication is at speed the counter weight has a tendency to fly out.

Anyway all good fun, sorry my build didn't work.  :-[

Ron


Back to the subject

https://youtu.be/eMGiCx4Yw0o

let me know if this URL doesn't work?

Ron
   
Group: Experimentalist
Hero Member
*****

Posts: 841
Back to the subject

https://youtu.be/eMGiCx4Yw0o

let me know if this URL doesn't work?

Ron

Just my thoughts on this but there are three states not two as in the PDF.

1) attract in

2) lock

3) repulse out

You can feel this with a couple of hand held magnets. There is a strong attract in force and in attempting  to move one of the magnets to the repulse position one is met with what I call a lock. One must move the magnet way out on the 45 degree line to allow movement to the repulse position. Then the magnet is so far away there is next to no repulsion.

If one positions the magnet further out with minimum attraction in then there is minimum attract in and minimum repulse out.

I have tried this with short magnets and longer length vs dia magnets and there seems to be no sweet spot.

23 views and no comments?

Ron
   

Group: Moderator
Hero Member
*****

Posts: 1856
Ron,
My ferris wheel idea was to keep the axes of the rotor magnets horizontal by using gravity, and I really meant the masses to be so heavy that magnetic forces couldn't alter that.  I see your rotor magnets are not kept horizontal, they flip about a lot.  Perhaps some other arrangement to keep those magnets always horizontal would be better.
Smudge
   
Group: Experimentalist
Hero Member
*****

Posts: 841
Ron,
My ferris wheel idea was to keep the axes of the rotor magnets horizontal by using gravity, and I really meant the masses to be so heavy that magnetic forces couldn't alter that.  I see your rotor magnets are not kept horizontal, they flip about a lot.  Perhaps some other arrangement to keep those magnets always horizontal would be better.
Smudge

Smudge,

True, but even holding the magnet horizontal does not change the outcome. Try it with two hand held magnets and you will see what I mean. Failing that, show a dimensioned model that works.

Edit: gee I hope that doesn't come through as being rude? But at the start of the video it shows almost no repulsion. At the end of the video it shows the lock point. And no matter how heavy you make the weights centrifigal force will still be present to oscillate the weights.

Ron
« Last Edit: 2020-04-05, 22:27:32 by ronee »
   

Group: Moderator
Hero Member
*****

Posts: 1856
Smudge,

True, but even holding the magnet horizontal does not change the outcome. Try it with two hand held magnets and you will see what I mean. Failing that, show a dimensioned model that works.

Edit: gee I hope that doesn't come through as being rude? But at the start of the video it shows almost no repulsion. At the end of the video it shows the lock point. And no matter how heavy you make the weights centrifigal force will still be present to oscillate the weights.

Ron
It doesn't come through as rude, and I am grateful that you are doing what you do.  Alas having moved house three times in the last two years I gave away all my magnets and bits and pieces to Grumage.  I am not convinced the idea will work but an FEMM simulation does show a small average torque in one direction, but that is much smaller than the peak torques that occur in both directions.  The pantograph method for keeping things parallel as seen on the link rod on steam locomotives could be one answer, see image below.
Smudge
   
Group: Experimentalist
Hero Member
*****

Posts: 841
It doesn't come through as rude, and I am grateful that you are doing what you do.  Alas having moved house three times in the last two years I gave away all my magnets and bits and pieces to Grumage.  I am not convinced the idea will work but an FEMM simulation does show a small average torque in one direction, but that is much smaller than the peak torques that occur in both directions.  The pantograph method for keeping things parallel as seen on the link rod on steam locomotives could be one answer, see image below.
Smudge

An analysis of the Ferris Wheel magnet motor:

In the first model with the rotor rotating at speed when the pivot is going CW up through the 9:00 position the weight nearly doubles. Then as it goes from left to right across the 12:00 position the weight lags behind but as the pivot goes down through 3:00 the weight is weightless and still traveling from left to right. At this point, 6:00, it is possible for the weight to do a full loop around the pivot.

However this is not the reason it doesn't work, mearly an indication that this design is limited to very low speeds.

As the north pole of the  rotor magnet  moves towards the south pole of the stator magnet there is attraction... an energy gain. However when the two magnets are edge to edge they so attract to each other so as to lock.  To continue the same rotation direction, energy must be provided to break the lock. Thus all the energy gained on attract in must be spent to overcome the lock.

But wait, there is more, as you slide the north pole down the length of the stator maget we have a situation where we have two like poles are approaching, a repulse situation so twice as much energy is required to continue rotation. This doesn't happen. as the rotation ceases at the lock position.

The two rotor concept, while clever, will suffer from the same restriction. Both versions have the same problem with alignment. When the rotor magnet is positioned in repell (if you could get it there) it is fully half a diameter above the stator magnet and the repell is nonexistant. You can see this in the video where at one point as I demonstate the weak repulsion I let the rotor go CCW and the the rotor magnet is attracted in and fired out in reverse. However it is then attacted back in to lock as at the end of the video.

Oh, do you like my tomato grow op in the background? I have 7 out in the green house, as they were getting too big.... frost this AM, ugh

Take care

Ron

   
Group: Elite
Hero Member
******

Posts: 3537
It's turtles all the way down
Just to fill in time while being socially isolated I looked back at Howard Johnson's work.  Here is my critique of his vortex claims, they are not what they seem.  My message is to ignore his ideas about PM fields.

That is not to say that the classical model for a PM is necessarily correct under all circumstances.  In a metal magnet conduction electrons become spin polarized and add to the magnetization.  Being mobile within the magnet they endure magnetic forces that can cause them to dynamically congregate in certain areas, thus the overall magnetization is no longer spatially uniform within the magnet.  I can't find any evidence that this effect has ever been considered, and this could be the key to making magnet motors work.

Smudge

Dear Smudge
Thank you for your paper on the Howard Johnson approach. I became interested in his approach back in the early 80's when it appeared on the cover of Popular Science magazine.

 (some pictures archived here: http://rexresearch.com/johnson/1johnson.htm )

I was deeply interested in the possibility of a all magnet motor and explored the many (non working) patents available for such devices. I should have suspected the Pop Science article was sensationalism to sell copies back then, but since Howard had a few patents, I gave him the benefit of the doubt and built a few variations of the cover device (motor) but they never worked to any degree.

 I came to the conclusion his measurements were flawed. He was only able to demonstrate linear motors that would propel for part of the cycle but never could reguage. He never was able to demonstrate a working rotary machine.

Such a device (magnet motor) could possibly be created, but I think it would work on a different principle than just manipulating repulsion and attraction as these always seem to null out, at least, in my experience. I believe it would have to use a deeper principle such as we find in many of your writings on the subject.

FWIW

Kind Regards
« Last Edit: 2020-04-07, 02:25:41 by ion »


---------------------------
"Secrecy, secret societies and secret groups have always been repugnant to a free and open society"......John F Kennedy
   

Group: Moderator
Hero Member
*****

Posts: 1856
Here's another paper for consideration by you all.  Essentially it is what is known as armature reaction in permanent magnet electric generators using commutators, where efficiency is improved by altering the angular position of the brushes to take account of a phase delay brought about by the L/R time constant of the inductance of the armature coil against the load resistance.   Heavier loading (reduced load resistance) creates greater armature reaction.  What is not recognized is the distinct possibility that if the brushes are not moved to compensate, and the efficiency gets very low, the lost energy that goes as heat can actually be overunity.  Taken to its limit, where no electrical output is taken because the load is virtually a short circuit, the device can produce more heat output than the mechanical drive input.  This has been demonstrated where the induced load currents are simply eddy currents in a rotating disc of electrically conductive material.  This paper explains how the armature reaction displaces the eddy current vortices that then has the effect of both reducing the driving torque and also creating closed electric field vortices above the surface of the disc that are seen by the stator magnets as unidirectional impulses that apply torque to load the spins responsible for the magnetism, hence explaining the source of excess energy.

Reference is also made to the Sweet and Manelas devices that could be solid state versions of this phenomenon.  More on that later.

Smudge 
   
Pages: [1] 2 3
« previous next »


 

Home Help Search Login Register
Theme © PopularFX | Based on PFX Ideas! | Scripts from iScript4u 2024-04-19, 21:33:14